Difference between revisions of "2021 Fall AMC 10B Problems/Problem 5"

m (Problem 5)
m (Solution 1)
Line 5: Line 5:
  
 
==Solution 1==
 
==Solution 1==
We have <cmath>n=8^{2022}=  \left(8^\frac{2}{3}\right)^{2022}=4^{3033}</cmath>
+
We have <cmath>n=8^{2022}=  \left(8^\frac{2}{3}\right)^{2022}=4^{3033}.</cmath>
Therefore, <cmath>\frac{n}4=\boxed{(\textbf{E})\:4^{3032}}</cmath>
+
Therefore, <cmath>\frac{n}4=\boxed{(\textbf{E})\:4^{3032}}.</cmath>
  
 
~kingofpineapplz
 
~kingofpineapplz

Revision as of 21:54, 22 November 2021

Problem 5

Let $n=8^{2022}$. Which of the following is equal to $\frac{n}{4}?$

$(\textbf{A})\: 4^{1010}\qquad(\textbf{B}) \: 2^{2022}\qquad(\textbf{C}) \: 8^{2018}\qquad(\textbf{D}) \: 4^{3031}\qquad(\textbf{E}) \: 4^{3032}$

Solution 1

We have \[n=8^{2022}=  \left(8^\frac{2}{3}\right)^{2022}=4^{3033}.\] Therefore, \[\frac{n}4=\boxed{(\textbf{E})\:4^{3032}}.\]

~kingofpineapplz